neurology-answers

.pdf

School

Lewis University *

*We aren’t endorsed by this school

Course

NURS-233

Subject

Medicine

Date

Apr 3, 2024

Type

pdf

Pages

47

Uploaded by SargentInternet11790 on coursehero.com

Part 2: Questions with Answers and Explanations Neuromuscular Medicine Questions Question 1 A previously healthy 68-year-old male presents with progressive muscle weakness in his right hand, swallowing difficulty and difficulty with walking. He believes these symptoms occurred “out of the blue” and have worsened over the last 2-3 months. His temperature is 98.6 deg F (37 deg C), blood pressure 145/86 mmHg, pulse is 80/min, and respirations are 17/min. Neurologic examination is significant for mild dysarthria. He has atrophy of the thenar muscles of the right hand and forearm, and 2/5 strength on wrist extension and flexion, as well as 3/5 power in intrinsic hand muscles on right. He has some fasciculations in his right thigh, and mild (4+/5) weakness of hip flexors bilaterally. Tone is increased in both legs. Sensory examination is intact. 3+ patellar reflexes are noted on the left lower extremity. On gait testing, the patient has poor balance, and spastic gait. Which of the following is the most likely diagnosis? A. Guillain-Barre syndrome B. Amyotrophic lateral sclerosis C. Myasthenia gravis D. Multiple sclerosis E. Dermatomyositis B - Amyotrophic lateral sclerosis: Correct Amyotrophic lateral sclerosis (ALS) is neurodegenerative disease that affects both upper- and lower-motor neurons. Therefore, patients affected by ALS will present with signs and symptoms of upper- and lower-motor neuron disease. Upper motor neuron (UMN) signs include: spasticity, positive Babinski, hyperreflexia, and spastic gait. LMN signs include: fasciculations (tongue fasciculations are highly specific), muscle atrophy, hyporeflexia, and steppage gait. It is important to remember that this is a motor neuron disease; therefore, there will be no loss of sensation. Most cases of ALS are sporadic, but there can be familial variants. Main cause of death is respiratory failure. Incorrect choices: A) Guillain-Barre syndrome (GBS) is the result of autoimmune destruction to myelin surrounding the peripheral nerves. Classically, patients present with ascending weakness, and decreased or absent deep tendon reflexes. C) Myasthenia gravis (MG) is a disease of the neuromuscular junction secondary to antibody- mediated destruction of acetylcholine receptors. This results in weakness. Patients with MG presents with fatigable muscle weakness. For example, patients may experience ptosis or diplopia when reading or watching television for a prolong period of time. D) Multiple sclerosis (MS) is an inflammatory disease of the central nervous system. Classically, the patient’s clinical presentation, and CNS lesions seen on MRI are disseminated in time and space. In
other words, autoimmune destruction of the CNS occurs in different regions at different times. Young adults are mostly affected, with a female predominance. E) Dermatomyositis is an inflammatory myopathy that presents as proximal muscle weakness, and characteristic dermatologic findings, such as Gottron papules, and a heliotrope rash Question 2 A 58-year-old female, accompanied by her husband, presents with trouble walking. She describes her legs as being heavy and stiff, and at times experiences muscle spasms in her lower extremities. Her husband notices that she has been having trouble writing, and buttoning her shirt. These symptoms have progressively worsened over that past 4-5 months. Neurologic exam is significant for intrinsic hand weakness, and arm fasciculations. The lower extremity is hypertonic, with increased patellar reflexes. On gait testing, the patient has poor balance, moves slowly, and has issues with turning. Which of the following is the best treatment to prolong life? A. Pyridostigmine B. Botulinum toxin C. Prednisone D. Riluzole E. Intravenous Immunoglobulin D - Riluzole: Correct This patient is presenting with signs and symptoms of both upper- and lower-motor neuron disease. This is highly suggestive of Amyotrophic Lateral Sclerosis (ALS), a neurodegenerative motor neuron disease. The main goal of treatment is to decrease the rate of neurodegeneration. Riluzole is the treatment of choice, as it is the only medication that positively affects survival in these patients. Incorrect choices : A) Pyridostigmine is an acetylcholinesterase inhibitor. This is effective in the management of Myasthenia gravis. ALS is secondary to neurodegeneration, not in deficiencies of acetylcholine. B) Botulinum toxin prevents the release of acetylcholine. It is a neurotoxin produced by Clostridium botulinum. It has a variety of uses, including treatment of cervical dystonia, and spasticity. It is not the best treatment for ALS. It can be used to treat refractory cases of sialorrhea in ALS. C) Prednisone is a glucocorticoid that has immunosuppressant properties. It can be used in the treatment of myasthenia gravis, multiple sclerosis, polymyositis, and dermatomyositis-to name a few. This is not involved in the management of ALS. E) Intravenous immunoglobulin is not involved in the management of ALS. This is a neurodegenerative disease, not an antibody mediated disease. Question 3
A 33-year-old female presents with drooping of her eyelid that seem to occur while reading or watching television. This appears to get worse later in the day. She also reports that at times she sees “double.” A few weeks ago, she was prescribed an antibiotic medication for a urinary tract infection. She does not recall the name of the antibiotic. On physical exam, bilateral ptosis, with the left affected more than the right is seen. Pupillary function intact. A glove is filled with ice, and subsequently applied to the patient’s eyelid. After two minutes, the patient’s ptosis has improved. Which of the following is most likely to yield rapid symptom improvement? A. Corticosteroids B. Plasma exchange C. Acetylcholinesterase inhibitor D. Thymectomy C - Acetylcholinesterase inhibitor: Correct This patient presents with fatigable muscle weakness (e.g., ptosis when reading or watching television,) and improvement of her ptosis after ice was applied to her eyelids. This is highly suggestive of myasthenia gravis (MG), an autoimmune disease of the neuromuscular junction. First- line treatment for symptomatic improvement in MG is with an acetylcholinesterase inhibitor (e.g., Pyridostigmine). This prevents the degradation of acetylcholine in the synaptic cleft, thus improving muscle weakness. Incorrect choices : A) Corticosteroids can be used in the treatment of MG. However, it is usually reserved for patients who do not adequately respond to acetylcholinesterase inhibitors. B) Plasma exchange can remove antibodies directed against acetylcholine receptors. This is typically used in certain circumstances, such as in myasthenic crisis. This patient is presenting with symptomatic ocular MG, and thus an acetylcholinesterase inhibitor is considered first-line. D) Thymectomy is performed in patients with a thymoma. This may increase the likelihood of remission from MG. This implies that the thymus may be involved in the development of MG. In fact, a CT or MRI scan of the mediastinum and chest is involved in the evaluation of suspected MG. Question 4 A 38-year-old female presents with muscle fatigue that is worse by the end of the day. She reports to have myalgias in the back of her neck, and at times her head “drops.” She also states to sees “double” when watching television, or reading for a prolonged period of time. On physical examination, ptosis is produced when asking the patient to look at the ceiling without blinking. Pupillary function is intact. Appropriate serologic testing is obtained. On repetitive nerve stimulation, the amplitude of the compound muscle action potential progressively declines. Which of the following is most likely the diagnosis? A. Botulism B. Lambert-Eaton myasthenic syndrome C. Acute inflammatory demyelinating polyradiculopathy D. Myasthenia gravis
E. Multiple sclerosis D - Myasthenia gravis: Correct Myasthenia gravis (MG) is an autoimmune disease of the neuromuscular junction. Antibodies are directed against acetylcholine receptors, resulting in muscle weakness. Fatigable muscle weakness is a cardinal feature of the disease. Ocular symptoms are very common; where patients may present with ptosis, as well as diplopia. There are a myriad of medications that can exacerbation symptoms in patients with MG. Therefore, one must use caution in using certain medications for other medical issues. These medications include aminoglycosides, neuromuscular blocking anesthetics, beta blockers, and so forth. The ice-pack test and Tensilon test are both sensitive beside tests for MG. Serologic testing can be used to confirm the diagnosis. Patients may have antibodies against acetylcholine receptors, or muscle specific tyrosine kinase. Incorrect choices: A) Botulism is a neurotoxin that results in cranial nerve palsies, and descending muscle weakness. A helpful clinical clue that can differentiate MG from botulism toxin is pupillary function. In botulism, pupils are dilated; in MG, pupils are normal. B) Lambert-Eaton myasthenic syndrome (LEMS) is a disease of the neuromuscular junction. It is the result of autoantibodies against voltage gated calcium channels on presynaptic neurons. This prevents the release of ACh, leading to muscle weakness. On repetitive nerve stimulation, there is an incremental increase in the compound muscle action potential. C) Acute inflammatory demyelinating polyneuropathy (AIDP) is the most common subtype of Guillain-Barre syndrome. This classically presents with ascending symmetric muscle weakness, and absent or decreased deep tendon reflexes. The electrophysiologic findings are more suggestive of MG. E) Multiple sclerosis (MS) is an autoimmune disease affecting the white matter of the central nervous system. Signs that are suggestive of MS are relapses and remissions. In other words, a patient may present with an isolated neurological deficit that later improves. Other signs include Lhermitte sign (flexion of the neck leading to a shock like sensation down the spine or limbs), intranuclear ophthalmoplegia, and Uhtoff phenomenon (worsening of neurological deficits with heat.) Question 5 A previously healthy 26-year-old male is brought to the emergency department due to muscle weakness. He reports mild weakness in his legs that has progressively worsened over a few days to the point that he can barely move his legs. He has some numbness in his legs and his arms feel weak. He has recently recovered from a respiratory infection. 1/5 muscle strength is noted throughout the lower extremities, as well as absent patellar and ankle reflexes. 3 to 4-/5 muscle strength is appreciated throughout his upper extremities. A lumbar puncture is performed, which shows the following: Cell count: 4 cells/mm3, Glucose: 67 mg/dL, Protein: 187 mg/dL Gram
stain: no organisms Spirometry is performed, which shows a forced vital capacity of 0.85 L. Which of the following is the best next step in management? A. Intravenous (IV) immunoglobulin B. Intubation C. IV corticosteroids D. Immunomodulating agent E. Plasma exchange B - Intubation: Correct Note the symmetric muscle weakness, and absence of deep tendon reflexes. This is highly suggestive of Guillain-Barre syndrome (GBS), especially in the setting of a recent respiratory infection (likely Mycoplasma pneumoniae). Another important finding is albuminocytologic dissociation, which describes a normal cell count and an elevated protein level (>45 mg/dL), which further supports the diagnosis. Although IVIG and plasmapheresis are the main treatments for GBS, it is important to realize this patient’s forced vital capacity is < 1L. This is an ominous sign of respiratory failure, and thus is an indication to intubate. This is true of other neuromuscular diseases that predispose patients to experience respiratory failure. Recall that other indications to intubate is a rapid decline in vital capacity, and a negative inspiratory pressure < 20 cm H2O. Question 6 A 30-year-old female presents with muscle weakness in the lower extremities and palpitations. Her symptoms have been progressively worse over the course of a 2 weeks. She reports to be in great health, but had a period of abdominal pain and diarrhea approximately 5 weeks ago. Her temperature is 98.9 deg F (37.2 deg C), blood pressure 117/82 mmHg, pulse is 102/min, and respirations are 16/min. On physical exam, there is 4-/5 muscle strength throughout the lower extremities. Bilateral patellar reflexes are 1+ and ankle reflexes are absent. Lumbar puncture was performed, and results are pending. Part 1: What is the most likely diagnosis? A. Polymyositis B. Guillain-Barre syndrome C. Hypokalemia D. Myasthenia gravis B - Guillain-Barre syndrome: Correct This patient is presenting with symmetric ascending muscle weakness, decreased deep tendon reflexes, and antecedent gastrointestinal infection (Campylobacter jejuni is most likely). This is highly suggestive of Guillain-Barre syndrome (GBS), and autoimmune disease of myelinated neurons in the peripheral nervous system. The mainstay of treatment is with plasma exchange or intravenous immunoglobulin. Both are equally effective, and do not have an additive effect when used together. Incorrect choices:
A) Polymyositis is an inflammatory myopathy that classically presents with symmetric proximal muscle weakness. Patients may complain of weakness when combing their hair, or when climbing a flight of steps. Treatment is with high-dose corticosteroids. C) Hypokalemia can be due to diarrhea, diuretic use, vomiting, hyperaldosteronism and so forth. Hypokalemia can result in muscle weakness, cramps, and rhabdomyolysis. Cardiac involvement may produce palpitations. Treatment involved replenishing potassium. D) Myasthenia gravis (MG) presents with fatigable muscle weakness. It results from autoimmune destruction of acetylcholine receptors on the sarcolemma. Pyridostigmine provides symptomatic relief in these patients. Part 2: What is the most appropriate initial treatment? A. High dose corticosteroids B. Intravenous immunoglobulin C. Potassium chloride D. Pyridostigmine B - Intravenous immunoglobulin: Correct This patient is presenting with symmetric ascending muscle weakness, decreased deep tendon reflexes, and antecedent gastrointestinal infection (Campylobacter jejuni is most likely). This is highly suggestive of Guillain-Barre syndrome (GBS), and autoimmune disease of myelinated neurons in the peripheral nervous system. The mainstay of treatment is with plasma exchange or intravenous immunoglobulin. Both are equally effective, and do not have an additive effect when used together. Incorrect choices: A) Polymyositis is an inflammatory myopathy that classically presents with symmetric proximal muscle weakness. Patients may complain of weakness when combing their hair, or when climbing a flight of steps. Treatment is with high-dose corticosteroids. C) Hypokalemia can be due to diarrhea, diuretic use, vomiting, hyperaldosteronism and so forth. Hypokalemia can result in muscle weakness, cramps, and rhabdomyolysis. Cardiac involvement may produce palpitations. Treatment involved replenishing potassium. D) Myasthenia gravis (MG) presents with fatigable muscle weakness. It results from autoimmune destruction of acetylcholine receptors on the sarcolemma. Pyridostigmine provides symptomatic relief in these patients. Question 7 A 44-year-old female presents with weakness. Her weakness becomes apparent when she picks up her youngest daughter, or when placing her nonperishable foods in the top shelf of her kitchen. She also reports difficulty when getting up from a chair. These symptoms began a few months ago, and have since worsened. On physical exam, there is an erythematous eruption on her eyelids, and erythematous papules on the dorsal aspect of her hands. Neurological examination is significant for
Your preview ends here
Eager to read complete document? Join bartleby learn and gain access to the full version
  • Access to all documents
  • Unlimited textbook solutions
  • 24/7 expert homework help